Đến nội dung

nguyenta98

nguyenta98

Đăng ký: 31-10-2011
Offline Đăng nhập: 16-01-2023 - 09:26
****-

#558636 ĐỀ THI OLYMPIC CHUYÊN KHOA HỌC TỰ NHIÊN NĂM 2015

Gửi bởi nguyenta98 trong 10-05-2015 - 17:59

Bài số học có thể giải như sau:

Xét một số nguyên tố $p$ bất kì $p\leq n$

TH1: $p|b$ thì $v_p(b^n.a.(a+b)...(a+(n-1)b)) \geq n$ trong khi $v_p(n!)=\frac{n-S_{p}(n)}{p-1}$ với $S_{p}(n)$ là tổng cs của $n$ trong cơ số $p$

Rõ ràng $v_p(n!)=\frac{n-S_{p}(n)}{p-1}<n$ nên ta có ngay $v_p(b^n.a.(a+b)...(a+(n-1)b))>v_p(n!)$

TH2: $(b,p)=1$ khi đó gọi $v_p(n!)=k$ thì do $(b,p)=1 \rightarrow (b,p^k)=1$ khi đó theo định lý Bezout, ta có tồn tại $c$ sao cho $bc \equiv 1 \pmod{p^k}$ thì do $(b,p)=1 \rightarrow (c,p)=1$

Khi đó ta cần cm

$$p^k|b^n.a.(a+b)...(a+(n-1)b)$$

$$\leftrightarrow p^k|a.(a+b)...(a+(n-1)b)$$

$$\leftrightarrow p^k|a.(a+b)...(a+(n-1)b).c^n$$

$$\leftrightarrow p^k|(ac+0.bc)(ac+1.bc)...(ac+(n-1)bc)$$

$$\leftrightarrow p^k|ac(ac+1)(ac+2)...(ac+n-1)$$ $(1)$

(do $bc \equiv 1 \pmod{p^k}$ )

Như vậy, ta có $\dfrac{ac(ac+1)(ac+2)...(ac+n-1)}{n!}=\binom{ac+n-1}{n}$ là số nguyên do đó $n!|ac(ac+1)(ac+2)...(ac+n-1)$ hay $(1)$ đúng vì $p^k||n!$

Như vậy qua cả 2 Th ta suy ra ngay với $p$ nguyên tố $p\leq n$ thì $p^{v_p(n!)}|b^n.a.(a+b)...(a+(n-1)b) \rightarrow n!|b^n.a.(a+b)...(a+(n-1)b)$ đpcm

 

P/S bài này đã từng có trong các giờ học đội dự tuyển và đội tuyển của KHTN khóa anh Hoàn, anh Đăng, đề năm nay quả thực có format rất giống thi quốc tế




#558625 ĐỀ THI OLYMPIC CHUYÊN KHOA HỌC TỰ NHIÊN NĂM 2015

Gửi bởi nguyenta98 trong 10-05-2015 - 16:28

Ai làm giùm bài hình với -_- ngồi cả buổi loay hoay mãi không ra -_-

Bài hình như sau: :D

a) Dễ cm các điều sau:

1) $\angle{AID}=\angle{DJA}=\dfrac{1}{2}.\angle{ABD}+90$ do đó $AIJD$ tg nội tiếp

2) Cộng góc suy ra tam giác $DJN$ đồng dạng tam giác $IAM$

3) $\angle{NJT}=\angle{AJI}=\angle{AID}=\angle{ADS}=\angle{ATS} \rightarrow \overline{I,J,P,Q}//ST$

4) Từ 3) suy ra ngay $\angle{NJT}=\angle{SAM}$

5) Dễ thấy 2 tam giác cân $TDJ$ và $SAI$ cân tại T,S và hai tam giác này đồng dạng nhau (cộng góc đơn giản)

6) Từ 2) và 5) suy ra hai cặp tam giác đồng dạng: $\Delta{JNT},\Delta{AMS}$ và $\Delta{TND},\Delta{SMI}$ (phép đồng dạng)

7) Từ 6) suy ra ngay $\angle{ASM}=\angle{NTJ}$ hay nếu $NT \cap SM = W$ thì $WAST$ là tg nội tiếp hay có đpcm

 

b) Bổ đề Sawayama mở rộng cho tứ giác:

Cho tứ giác $ABCD$ nội tiếp $(O)$, $I,J,M,N$ vai trò tương tự như trong đề bài thì khi đó đường tròn $(S_{1})$ tx trong $(O)$ và tiếp xúc $AB,CD$ thì nó cũng tiếp xúc $(O),AB,CD$ tại tương ứng $W,M,N$ (dành cho các bạn cm, nếu cần mình sẽ post, cũng chú ý thêm là có 2 dg tròn thỏa mãn tx trong $(O)$ và tx $AB,CD$, phụ thuộc vào vị trí nằm của nó mà ta cm)

 

Quay lại câu b, xét dg tròn $(S_{2})$ tx trong $(O)$ và tx $AB,CD$ đồng thời $ (S_{2})$ khác $(S_{1})$ thì nó tx $(O),AB,CD$ tại $K,Q',P'$

Khi đó ta có $AB \cap CD = G$ thì $G$ là tâm vị tự ngoài $(S_{1}),(S_{2})$ và $K,W$ là tâm vị tự ngoài của $(S_{2}),(O)$ và $(S_{1}),(O)$ tương ứng, khi ấy theo định lý D' Alambert ta có $K,G,W$ thẳng hàn, và tương tự câu a kết hợp bổ đề nêu trên ta cũng có $SQ' \cap TP' = K$ và $P'Q'//ST$ và $P'Q'$ đi qua tâm nội tiếp tam giác ABC,DBC (*)

Do đó

$\angle{SQ'M}=\angle{KQ'B}=\angle{Q'P'K}=\angle{STK}=\angle{SWK}$ (do sự tiếp xúc) và suy ra ngay $MQ'KW$ nội tiếp, tương tự $NP'KW$ nội tiếp

Suy ra $GM.GQ'=GK.GW=GC.GD$ (do $G,K,W$ thẳng hàng cmt) suy ra $MQ'CD$ nội tiếp hay $Q'$ trùng $Q$ tương tự $P'$ trùng $P$ (**)

Từ (*)(**) suy ra đpcm

 

 

P/S Một kiểu khác, ta có thể cm $MQ'KW$ nội tiếp như sau: $SM.SW=SA^2=SB^2=SQ'.SK$




#558463 ĐỀ THI OLYMPIC CHUYÊN KHOA HỌC TỰ NHIÊN NĂM 2015

Gửi bởi nguyenta98 trong 09-05-2015 - 15:48

Lời giải bài hình (mình loay hoay cả trưa không up dc cái hình mọi ng thông cảm)

a) Ta có $AD$ giao $(ABC)$ tại $X$ Từ $D$ kẻ dg thẳng vg với $AD$ cắt $(IBC)$ tại $U,V$  thì do $XD.XA=XB^2$ do đó $AU,AV$ tiếp xúc $(IBC)$

Từ đó suy ra cái tg PUQV điều hòa từ đó có ngay tính chất quen thuộc $DP.DQ=DV.DU=DU^2=DB.DC=DX.DA$ ko đổi

b)$XR$ giao $(ABC)$ tại $W$ thì $XR.XW=XB^2=XD.XA$ do vậy $W$ thuộc $(ADR)$ hay $AW$ là trục đp của $(ADR),(ABC)$

Mà $N=ST \cap BC$ do đó $N$ là tâm đẳng phương 3 đg tròn $(ABC),(ADR), ( IBC)$ do đó $A,W,N$ thẳng hàng

Ta lại có $\angle{XMD}=\angle{XRM}=\angle{XAW}$ (do $DRWA$ tg nội tiếp)

Nên $AMXN$ tg nội tiếp

suy ra $DM.DN=DA.NX$ mặt khác $DB.DC=DP.DQ=DM.DQ$( do câu a))

Suy ra $DN=DQ$ đpcm




#558436 ĐỀ THI OLYMPIC CHUYÊN KHOA HỌC TỰ NHIÊN NĂM 2015

Gửi bởi nguyenta98 trong 09-05-2015 - 11:35

Bài số học năm nay khá nhẹ nhàng:

Bài 1: (vắn tắt) $3^p+4^p=x^2 \rightarrow 3^p=(x-2^p)(x+2^p) \rightarrow x-2^p=3^m,x+2^p=3^n \rightarrow 2^{p+1}=3^n-3^m \rightarrow m=0,n=p$ (xét mod(3))

Suy ra ngay được $2^{p+1}=3^p-1$ nếu $p>3$ dùng Fermat bé suy ra ngay vô lý, còn $p\le 3$ thì $p=2$ thỏa mãn

 

Bài 3: Quy ước 1x3 là quân nằm ngang và 3x1 nằm dọc

Chia bàn cờ ra thành $671$ cụm 3x3 và một cụm 3x2, gọi các cụm này là $C_1,.,C_{671}$, riêng cụm 3x2 không quan tâm, ta khẳng định bạn $A$ thắng như sau: đầu tiên $A$ đặt quân 1x3 vào $C_1$ (có thể nằm ở hàng trên cùng, giữa hoặc cuối của $C_1$ không quan trọng vì sau khi đặt vào thì $B$ ko thể đặt quân 3x1 nào vào cụm đó nữa) sau đó do bạn $B$ chỉ đặt 3x1 (tức quân nằm dọc) giả sử quân đó thuộc $C_i$ nào đó thì $A$ chỉ cần không đặt quân 1x3 bị chèn vào $C_i$ đó, mà cụ thể $A$ đặt 1x3 vào một $C_j$ khác với $j$ khác $i$, khi đó ta có ngay được $A$ sẽ luôn chiếm giữ được ít nhất $\dfrac{671+1}{2}=336$ cụm 3x3 trong khi $B$ chỉ chiếm giữ được tối đa $335$ cụm 3x3 (do A đi trước), như vậy việc còn lại là hoàn thành các cụm 3x3 thì $A$ có thể hoàn thành với ít nhất $336x3=1008$ quân 1x3 trong khi $B$ chỉ hoàn thành tối đa $2015-1008=1007$ quân 3x1 (cho dù $B$ chiếm giữ cụm 3x2 thừa ra thì vẫn chỉ thu được tối đa 1007 quaan3x1)  tức là $A$ thắng (vì đến lúc B sẽ không đặt được nữa trước khi $A$ không đặt được !)

 

P/S qua cách giải này ta có thể thấy nếu bàn cờ 3xn với $n=6k+t$ với $t$ lẻ thì $A$ sẽ thắng




#485028 Đề thi học sinh giỏi toán lớp 10 THPT Chuyên ĐHSP HN

Gửi bởi nguyenta98 trong 27-02-2014 - 16:48

Bài 4.
Trong một hội nghị có $n$ người tham dự mà ta kí hiệu là $A_1,A_2,...,A_{n}$ ($n\geq 4$, $n\in\mathbb{Z}$). Biết rằng 2 người bất kì không quen nhau có đúng 2 người quen chung trong hội nghị. Giả sử rằng $A_1,A_2$ quen nhau và không có người quen chung trong hội nghị. Chứng minh rằng số người quen $A_1=A_2$

Ý tưởng chung là xây dựng song ánh

Giải như sau:

Kí hiệu $Q(A_1),Q(A_2)$ là tập người quen của $A_1,A_2$ tương ứng

Xét một người $x \in S(A_1)$ rõ ràng $x$ không quen $A_2$ (do giả thiết bài toán là $Q(A_1)\cap Q(A_2)=\phi$) do đó theo đk đề bài thì $x$ và $A_2$ cùng quen đúng 2 người khác, một trong hai người đó chính là $A_1$ và người còn lại là $y$, mà $A_2$ quen $y$ nên $y \in Q(A_2)$

Như vậy mỗi người $x$ thuộc $S(A_1)$ thì tồn tại duy nhất một người $y \in Q(A_2)$ quen $x$ và lập luận tương tự với $y$ thì có duy nhất $x$ thuộc $Q(A_1)$ mà quen $y$ do đó có một song ánh từ $f: Q(A_1)\rightarrow Q(A_2)$ nên lực lượng hai tập phải bằng nhau, đpcm

P/S

 

Bài 1.
Cho 4 số thực $a,b,c,d$ thỏa mãn $a+b+c+d=2$. Chứng minh rằng :
1. $a^2-a+1\leq \frac{3}{4}(a^2+b^2+c^2+d^2)$
2. $\frac{a}{a^2-a+1}+\frac{b}{b^2-b+1}+\frac{c}{c^2-c+1}+\frac{d}{d^2-d+1}\leq \frac{8}{3}$

 

Còn bài BDT mình làm thế này

áp dụng cô si ngược dấu ta cần cm

$\sum{1-\dfrac{a}{a^2-a+1}}\geq 4-8/3=4/3$

$\Leftrightarrow \sum{\dfrac{a^2-2a+1}{a^2-a+1}}\geq 4/3$

Nhận thấy $a^2-2a+1=(a-1)^2\geq 0$ và tương tự với $b,c,d$

Do đó $\sum{\dfrac{a^2-2a+1}{a^2-a+1}}\geq \dfrac{a^2-2a+1+b^2-2b+1+c^2-2c+1+d^2-2d+1}{\frac{3}{4}.(a^2+b^2+c^2+d^2)}=\dfrac{a^2+b^2+c^2+d^2-2(a+b+c+d)+4}{\frac{3}{4}(a^2+b^2+c^2+d^2)}=4/3$

nên có đpcm

dấu đẳng thức khi dấu đẳng thức ở câu $a$ và đk xảy ra hay $a=b=c=d=1/2$




#475261 [VMO 2014] Ngày 2 - Bài 7 - Tổ hợp

Gửi bởi nguyenta98 trong 04-01-2014 - 17:05

Đúng vậy, trên chỉ là ý tưởng giải thôi anh, còn nếu xét đầy đủ thì phải xét là trong đống trên có số 0 nào hay không, nếu có số 0 thì phải xét riêng ra và ra đáp số như anh nói, ngoài ra nếu không có bất kì số 0 nào trong đó thì mới làm đoạn ý tưởng trên :D




#475224 [VMO 2014] Ngày 2 - Bài 7 - Tổ hợp

Gửi bởi nguyenta98 trong 04-01-2014 - 13:37

Bài 7 (6,0 điểm). Tìm tất cả các bộ số gồm $2014$ số hữu tỉ không nhất thiết phân biệt, thỏa mãn điều kiện: nếu bỏ đi một số bất kì trong bộ số đó thì $2013$ số còn lại có thể chia thành $3$ nhóm rời nhau sao cho mỗi nhóm gồm $671$ số và tích tất cả các số trong mỗi nhóm bằng nhau.

Giải như sau: (ý tưởng giải)

Trước tiên ta nhận thấy rằng có thể cho rằng cả 2014 số này nguyên, thật vậy nếu chúng hữu tỉ thì ta quy đồng mẫu số và được phép bỏ mẫu số đi thì điều kiện của bài toán không hề thay đổi, do đó ta thực hiện quy đồng để quy bài toán cho lớp số nguyên

Gọi $a_1,a_2,...,a_{2014}$ là các số thoả mãn đề bài và $a_i$

Cũng chú ý nếu $a_1,a_2,...,a_{2014}$ thoả đề thì ta thấy $|a_1|,|a_2|,...,|a_{2014}|$ cũng thoả mãn, cho nên ta có thể coi rằng $a_i \geq 0$ với mọi $i=1,2014$

Xét một số nguyên tố $p \in P$ bất kì, đặt $m_i=v_p(a_i)$ với mọi $i=1,2,...,2014$

Nếu $a_1,a_2,...,a_{2014}$ thoả mãn đề bài khi ta bỏ một số bất kì thì $2013$ số còn lại tách thành 3 tập có $671$ phần tử có tích bằng nhau, nghĩa là $v_p$ của ba tập đó cũng phải bằng nhau, điều này có nghĩa là tập $m_1,m_2,...,m_{2014}$ thỏa mãn điều kiện sau đây:

$i)$ $m_i \in N$ với mọi $i=1,2,...,2014$

$ii)$ bỏ bất kì phần tử nào của $m_1,m_2,...,m_{2014}$ thì tập còn lại gồm $2013$ số nguyên không âm sao cho có thể phân hoạch thành 3 tập con 671 phần tử có tổng bằng nhau

Như vậy đặt $S=m_1+m_2+...+m_{2014}$ thì $S-m_i$ có thể phân hoạch thành $A,B,C$ đôi một rời nhau sao cho $S(A)=S(B)=S(C)$ với $S(X)$ là tổng các phần tử của $X$

Có nghĩa là $S-m_i \vdots 3$ với mọi $i=1,2,3,...,2014$ hay $S \equiv m_1 \equiv m_2 \equiv m_3 ...\equiv m_{2014} \pmod{3}$
Đặt $m_i=3k_i+r$

Nhận thấy nếu $m_1,m_2,..,m_{2014}$ thỏa mãn $i)$ và $ii)$ thì tập $k_1,k_2,...,k_{2014}$ cũng như vậy, chúng cũng đồng dư nhau modulo 3 và lại lùi, lập luận tương tự cho bộ $k_1,k_2,...,k_{2014}$ và dùng lùi vô hạn suy ra mọi số $m_1,m_2,...,m_{2014}$ phải bằng nhau (có thể dùng $v_3$ thay cho việc lùi vô hạn cũng được) từ $m_1,m_2,...,m_{2014}$ bằng nhau suy ra $v_p(a_1)=v_p(a_2)=...=v_p(a_{2014})$ cho $p->\infty$ ta có ngay $a_1=a_2=...=a_{2014}$ tuy nhiên đây là $a_1,a_2,...,a_{2014}$ nguyên không âm, còn việc âm thì xét dấu dành cho các bạn :)

 

P/S, lâu lắm mới thấy VMO cho lại dạng bài kiểu sum of set thế này




#464628 Korea Final round 2010

Gửi bởi nguyenta98 trong 16-11-2013 - 15:34

Nhác post quá, các bạn xem lời giải và bình luận qua hình sau nhé :D (P/S chú Tuấn Anh biết anh chụp quyển nào chứ :D)

IMG_20131116_153139.jpg

IMG_20131116_153205.jpg




#459330 Đề chọn đội tuyển thi Quốc Gia Khối chuyên ĐHSP 2013-2014

Gửi bởi nguyenta98 trong 22-10-2013 - 21:57


Câu 2.

Với mỗi số nguyên dương $k$ ta đặt $f(k)=6^k+9^k+10^k+15^k$. Một cặp 2 số nguyên dương $(m;n)$ được gọi là cặp đôi hạnh phúc nếu $f(m)$ và $f(n)$ cùng chia hết cho $mn$.

1, Cho 2 số nguyên lẻ $m,n>1$ và $(m,n)$ là 1 cặp đôi hạnh phúc. Chứng minh rằng $25|mn$ nhưng $125 \not | mn$.

2, Chứng minh rằng tồn tại số nguyên dương $k$ để $(k,k)$ là cặp đôi hạnh phúc và nếu phân tích $k=p_1^{\alpha_1}....p_r^{\alpha_r}$ thì $\alpha_1+...+\alpha_{n}=1911^{103}$. Ở đây $p_1,p_2,...,p_r$ là các số nguyên tố đôi 1 phân biệt và $\alpha_{1},...\alpha_{r}$ là các số nguyên không âm.


 

Giải như sau:

Câu 2: 1) $(2^n+3^n)(3^n+5^n) \vdots mn$ và tương tự với $m$
Ta đặt $p$ là ước nguyên tố nhỏ nhất của $n$ có hai khả năng

Nếu $p|2^n+3^n \rightarrow p|2^{2n}-3^{2n}$ gọi $a$ nhỏ nhất sao cho $2^a-3^a \vdots p$ nên $a|2n$ mà $a$ phải chẵn vì $a$ lẻ thì $a|n$ khi đó $2^n-3^n \vdots 2^a-3^a \vdots p \rightarrow 2.3^n \vdots p$ mâu thuẫn, do đó $a=2k$ suy ra $k|n$ nên $k$ lẻ do đó $2k|p-1$ và $k$ lẻ nên $k=1$ vì nếu $k>1$ thì tồn tại $r$ lẻ, $r|k<p-1<p \Rightarrow r<p$ mà $r|n$ mâu thuẫn do đó $k=1$ nên $p=5$ suy ra $5|n$

Nếu $p|3^n+5^n$ lập luận tươnt tự suy ra $p|3+5=8$ vô lí vì $p$ lẻ do $p|n$ lẻ

Do đó ta cm xong $5|n$ và tương tự với $m$ đặt $5^x||n,5^y||m$ theo LTE, $2^n+3^n \vdots 5^{x+y} \Rightarrow 2^{5^x}+3^{5^x} \vdots 5^{x+y} \Rightarrow v_5(2+3)+v_5(5^x)\geq x+y \Rightarrow 1+x\geq x+y \Rightarrow y\le 1$ mà $y\geq 1$ nên $y=1$ tương tự $x=1$ từ đó có $25|mn,mn \not \vdots 125$

2) Từ giả thiết có $f(k) \vdots k^2 \Rightarrow (2^k+3^k)(3^k+5^k) \vdots k^2$

Ta xây dựng $k$ theo cách sau:

Quy nạp điều sau: Tồn tại $k$ sao cho $k=5.p_2...p_l$ với $p_i \neq p_j$

Và $p_i|2^{5.p_2...p_{i-1}}+3^{5.p_2...p_{i-1}}$

Đầu tiên $k=5.x_1$ ($p_1=5$) tiếp đó $k=5.p_2.x_2$ với $p_2|2^5+3^5$ ($p_2\neq 5$ hay $p_2=11$)

Giả sử quy nạp ta đã có $k=5.p_2.p_3...p_l$ với $p_i|2^{5.p_2...p_{i-1}}+3^{5.p_2...p_{i-1}}$ các số $p_i$ đôi một pb
Ta cm tồn tại $k$ để $k=5.p_2.p_3...p_l.p_{l+1}$

Thật vậy ta có $2^k+5^k=2^{5.p_2.p_3...p_l}+3^{5.p_2.p_3...p_l}$

Đặt $2^{5.p_2.p_3...p_{l-1}}=A,B=3^{5.p_2.p_3...p_{l-1}}$
Khi đó $A^{p_l}+B^{p_l}=(A+B)(A^{p_l-1}-...+B^{p_l-1})$

Chú ý theo cách chọn $p_i|2^{5.p_2...p_{i-1}}$ do đó $5,p_2,...,p_l|2^{5.p_2.p_3...p_{l-1}}+3^{5.p_2.p_3...p_{l-1}}=A+B$

Mặt khác $gcd((A+B);(A^{p_l-1}-...+B^{p_l-1})|p_l$ $(1)$ trong khi $A+B \vdots p_l$ nên $A^{p_l-1}-...+B^{p_l-1} \vdots p_l$ mặt khác $v_{p_l}(A^{p_l}+B^{p_l})=v_{p_l}(A+B)+v_{p_l}(p_l)=v_{p_l}(A+B)+1$ do đó $v_p(A^{p_l-1}-...+B^{p_l-1})=1$ mà $(A^{p_l-1}-...+B^{p_l-1})>p_l$ do đó nó còn có một ước nguyên tố khác $p_l$ và khác hẳn $5,p_2,...,p_{l-1}$ (do $(1)$) gọi ước nguyên tố đó là $p_{l+1}$

Do đó $k=5.p_2...p_l.p_{l+1}$ dễ kiểm chứng đúng nên quy nạp đúng

Như vậy từ cách quy nạp trên ta có thể thấy tồn tại $k$ là số square-free tức là số có dạng $p_1p_2...p_l$ với $l$ tuỳ ý

Cũng thấy rằng $k=p_1p_2...p_l$ và $2^k+3^k \vdots k$ với $p_1,p_2,..,p_l$ được chọn theo phương thức như cách quy nạp thì ta thấy $2^k+3^k=2^{p_1p_2...p_l}+3^{p_1p_2...p_l}$ thấy $2^{p_1..p_i}+3^{p_1...p_i} \vdots p_{i+1}$ (theo phương thức chọn quy nạp) do đó $v_{p_{i+1}}(2^{p_1p_2...p_l}+3^{p_1p_2...p_l})=v_{p_{i+1}}(2^{p_1..p_i}+3^{p_1...p_i})+v_{p_{i+1}}(p_{i+1}...p_l)\geq 1+1=2 \Rightarrow \vdots p_{i+1}^2$

(với mọi $i=\overline{1,l}$) từ đây suy ra $2^k+3^k \vdots k^2$

Chọn $l=1911^{103}$ khi đó $\alpha_1+...+\alpha_l=1911^{103}$ đây là đpcm

P/S bài này cho nhiều số $6,9,10,15$ để đánh lạc hướng chứ thực ra chỉ cần đề bài là $2^n+3^n \vdots n^2$ là được




#459212 Đề chọn đội tuyển thi Quốc Gia Khối chuyên ĐHSP 2013-2014

Gửi bởi nguyenta98 trong 22-10-2013 - 15:06

Câu 4.

Cơ sở dữ liệu tạp chí của thư viện Quốc Gia có đúng 2016 loại khác nhau . Thư viện này cho phép 2013 thư viện địa phương kết nối để có thể khai thác cơ sở dữ liệu tạp chí của nó. Biết mỗi thư viện địa phương được phép khai thác ít nhất 1008 loại tạp chí khác nhau và 2 thư viện địa phương bất kì có tối đa 504 loại tạp chí mà cả 2 thư viện địa phương đó cùng đc phép khai thác. Chứng minh rằng không có quá 1 loại tạp chí trong cơ sở dữ liệu của thư viện Quốc Gia mà cả 2013 thư viện địa phương đều không thể khai thác được

Đây là câu hay nhất trong đề, ý tưởng là đếm bằng hai cách

Giải như sau:

Giả sử phản chứng có hai loại tạp chí mà không có bất kì một thư viện địa phương nào khai thác nó. Ta chuyển bài toán về ngôn ngữ tập hợp:
Bài toán: Có tồn tại hay không 2013 tập con $A_1,A_2,...,A_{2013} \subset X=\{1,2,3,...,2014\}$ sao cho

$i)$ $|A_i \cap A_j| \le 504$

$ii)$ $|A_i|\geq 1008$

Mục đích của ta là chứng minh không tồn tại thì khi ấy giả sử phản chứng sẽ sai hay bài toán được cm

Để cm ta cần thực hiện đếm bằng hai cách hai lần cho phép hợp và phép giao

1) Đặt $M=\sum{|A_i \cup A_j|}$ rõ ràng $M=\sum{|A_i \cup A_j|}=\sum{|A_i|+|A_j|-|A_i\cap A_j|}\geq (1008+1008-504).\binom{2013}{2}$ $(1)$

Gọi $a_i$ là số tập con trong số $A_1,...,A_{2013}$ nhận $i$ làm phần tử của nó

Khi đó ta đễm $M$ theo cách hai như sau:

$M=\sum_{i=1}^{2014}{\left(\binom{a_i}{2}+(2013-a_i)a_i\right)}$ $(2)$ thật vậy $\binom{a_i}{2}$ là số cặp $A_m,A_n$ để $a_i$ cùng nằm trong cả $A_m,A_n$ còn $(2013-a_i)a_i$ là số cặp $A_m,A_n$ để $a_i$ nằm vào một trong hai $A_m,A_n$ (chú ý là định nghĩa của phép hợp là nếu $x \in |A \cup B|$ thì $x$ thuộc A hoặc B hoặc có thể cả $A,B$ cũng được)

Do đó từ $(1)(2)$ suy ra $\sum_{i=1}^{2014}{\binom{a_i}{2}+(2013-a_i)a_i}\geq (1008+1008-504).\binom{2013}{2}$

$$\Leftrightarrow \sum_{i=1}^{2014}{\dfrac{4015a_i-a_i^2}{2}}\geq 1512.\binom{2013}{2}$$

$$\Leftrightarrow \dfrac{4015(\sum_{i=1}^{2014}{a_i})-\sum_{i=1}^{2014}{a_i^2}}{2}\geq 1512.\binom{2013}{2}$$

Áp dụng BDT cauchy $\sum_{i=1}^{2014}{a_i^2}\geq \dfrac{1}{2014}(\sum_{i=1}^{2014}{a_i})^2$
Ta có $\dfrac{4015(\sum_{i=1}^{2014}{a_i})-\dfrac{1}{2014}(\sum_{i=1}^{2014}{a_i})^2}{2}\geq \dfrac{4015(\sum_{i=1}^{2014}{a_i})-\sum_{i=1}^{2014}{a_i^2}}{2}\geq 1512.\binom{2013}{2}$

Đặt $\sum_{i=1}^{2014}{a_i}=S$
Suy ra $\dfrac{4025S-\dfrac{1}{2014}S^2}{2}\geq 1512.\binom{2013}{2} \Leftrightarrow 0\geq \dfrac{1}{2014}S^2-4025S+3024.\binom{2013}{2}$ $(*)$
Theo định lý về tam thức bậc hai, $x_1\le S\le x_2$ với $x_1,x_2$ là hai nghiệm nhỏ lớn của $(*)$

Thử trực tiếp ta có $2029609\le S\le 6076471$ $(A)$

$$**********$$
2) Đặt $N=\sum{|A_i\cap A_j|}$ tương tự trên, ta dễ dàng cm được $N\le 504.\binom{2013}{2}$
Mặt khác cũng tương tự, dễ cm $N=\sum_{i=1}^{2014}{\binom{a_i}{2}}$

Do đó $504.\binom{2013}{2}\geq \sum_{i=1}^{2014}{\binom{a_i}{2}}$

$$\Leftrightarrow 504.\binom{2013}{2}\geq \sum_{i=1}^{2014}{\dfrac{a_i^2-a_i}{2}}$$

$$\Leftrightarrow 504.\binom{2013}{2}\geq \dfrac{\sum_{i=1}^{2014}{a_i^2}-S}{2}$$

Áp dụng BDT cauchy lần nữa $\sum_{i=1}^{2014}{a_i^2}\geq \dfrac{1}{2014}S^2$
Do đó $504.\binom{2013}{2}\geq \dfrac{\dfrac{1}{2014}.S^2-S}{2}$

$\Leftrightarrow 1008.\binom{2013}{2}\geq \dfrac{1}{2014}.S^2-S$
$\Leftrightarrow \dfrac{1}{2014}.S^2-S-1008.\binom{2013}{2} \le 0$
Giải BPT bậc hai này tương tự trên ta có $0\le S\le 2028600$ $(2)$ (do $S \in N$) $(B)$

Từ $(A),(B)$ suy ra $2029609\le S\le 2028600$ điều này mâu thuẫn do đó giả sử phản chứng sai nên bài toán cm xong :)

Ở đây ta thấy hai số $2029609;2028600$ chặt với nhau một cách khá ấn tượng, cho nên nếu làm mạnh bài toán thành cmr tồn tại hai loại tạp chí thì chưa chắc đúng :D




#455640 Chứng minh định lý Lớn Fermat với kiến thức PT

Gửi bởi nguyenta98 trong 06-10-2013 - 14:45

   Bạn hãy xem lại từng việc thu gọn ở 1) tr4,  2) tr5,  5) tr5,  6) tr6 và 8)tr6 thì sẽ thấy rõ, còn các 3), 4), 7) thì không cần thiết vì luôn chia hết cho $a^{2}$. Mình nghĩ làm mình làm đúng ở các chổ nói trên, nên có kết quả vậy.

   Mình đã thử với khai triển trực tiếp khi n = 5 ứng với u, v, t tìm được thì thấy trùng với kết quả thu gọn theo cách tổng quát của mình nhưng quá dài và phức tạp lắm.

   Bạn có thể kiểm tra dùm mình theo cách này không?.

Chào thầy,

Em thực sự cảm phục sự nỗ lực của thầy, tuy nhiên do trình độ có hạn cộng với hiểu biết chưa sâu sắc nên em không thể nói trước được cm của thầy đúng hay không, tuy nhiên em có một đề xuất, nếu thầy hoàn toàn tự tin về cách cm của mình thì thầy hãy cố gắng dịch phần nội dung văn bản sang tiếng anh rồi up lên các trang như sau:

1) http://arxiv.org/ cụ thể là vào mục Mathematics -> Number theory ở đây  http://arxiv.org/list/math.NT/recent

(trang này đã có nhiều nhà toán học up lên và được giải Field như Perelman, Terrence Tao)

2) http://math.stackexc...e.com/questions

3) http://www.artofprob...wforum.php?f=56

Chúng ta hãy chờ xem cộng đồng toán học quốc tế phản ánh ra sao?




#455639 tồn tại một bội $a(n)$ của $2^n+1$

Gửi bởi nguyenta98 trong 06-10-2013 - 14:34

Chứng minh với mọi số nguyên dương $n$, luôn tồn tại một bội $a(n)$ của $2^n+1$ sao cho: $a(n)$ có đúng $n$ số $1$, và $n$ số $1$ này đứng liên tiếp.

Ví dụ: Có thể chọn

$$a(1)=12; a(2)=110; a(3)=456111$$

Bài này tuy lời giải ngắn gọn, nhưng ý tưởng và kĩ năng ko phù hợp THCS  lắm :D Nên chuyển nó vào mục Olympiad

Giải như sau:

Ta có một chú ý nhỏ là nếu $2^n+1=5^x.y$ với $gcd(5,y)=1$ thì trong bội của $a(n)$ ta chỉ việc thêm $x$ số $0$ vào sau là xong, do đó để thuận tiện ta chỉ cm bài toán trong TH $gcd(2^n+1,5)=1$ là đủ

Xét số có dạng sau: $A=\overline{200..00200..00200..200...011111....1}$ với $n$ số 1 với $l$ chữ số 2, các chữ số $0$ ko quan tâm

Hay ta viết $A$ dạng sau:

$$A=2.10^{m_1}+2.10^{m_2}+...+2.10^{m_l}+11...11$$ (với $n$ cs $1$)

Chú ý $gcd(10,2^n+1)=1$ do đó chọn $m_i=\phi(2^n+1).s_i$ với $s_i \in Z+$ và $s_1>s_2>...>s_l$

Khi đó theo định lý Euler

$A \equiv 2+2+...+2+11...11 \pmod{2^n+1}$ (n cs 1 và $l$ cs 2)
Hay $A \equiv 2l+11...11 \pmod{2^n+1}$

Đến đây do $gcd(2,2^n+1)$ theo bổ đề Bezout, tồn tại $l$ để $2l+11..11 \vdots 2^n+1$ nên bài toán được cm, số $A \vdots 2^n+1$ và có đúng n cs 1

P/S bài có ý tưởng khá giống một bài Shorlist với cách chọn phi hàm Euler (bài $S(n) \vdots k$ gì đó mình ko nhớ lắm), nhắc lại hệ quả của bổ đề Bezout (thực ra gọi thế cho oai chứ nó là hệ thặng dư) Cho a,b,c nguyên dương, $gcd(a,c)=1$ khi đó tồn tại duy nhất số tự nhiên $x$ theo $\pmod{c}$ để $ax+b \vdots c$




#447971 Chứng minh $p|(x+1)(x^2+1)(x^3+1) \cdots (x^{q-1}+1)-1...

Gửi bởi nguyenta98 trong 05-09-2013 - 13:56

Trước hết bằng phương pháp quy nạp ta tính được :

                             $\prod_{k=1}^{q-1}(x^{k}+1) -1=\sum_{k=0}^{\frac{q(q-1)}{2}-1}x^{\frac{q(q-1)}{2}-k}$

Dòng này hình như có vấn đề đấy bạn, :D thử xem lại nhé, khai triênt cái này không đơn giản như vậy, theo mình là thế, hơn nữa mỗi phần tử dạng $x^{\frac{q(q-1)}{2}-k}$ có thể lặp nhiều lần (phụ thuộc vào k) nữa




#438050 IMO 2013

Gửi bởi nguyenta98 trong 25-07-2013 - 11:25

 

Bài 2. Trên mặt phẳng cho $2013$ điểm màu đỏ và $2014$ điểm màu xanh, trong đó không có ba điểm nào thẳng hàng. Ta chia mặt phẳng bởi các đường thẳng (không đi qua bất kì điểm nào trong các điểm đã cho) thành các vùng, sao cho không có bất kì vùng nào chứa các điểm có hai màu khác nhau. Hỏi cần ít nhất là bao nhiêu đường thẳng để luôn thực hiện được cách chia đó ?

 

 

Mình thử giải, các bạn góp ý nhé

Giải như sau:

Trước tiên ta cm bổ đề 1: Cho $a$ điểm xanh và $a$ điểm đỏ cách đều nhau trên đường tròn và sắp xếp xen kẽ (tức một xanh lại một đỏ), thi số đường thẳng ít nhất để chia thành các vùng thỏa mãn điều kiện đề bài là $a$

Chứng minh:
Nếu ta coi một điểm xanh và một điểm đỏ kề nhau là một "cụm" thì tồn tại một đường thẳng đi qua giữa hai điểm xanh đỏ của cụm đó mà ta gọi tạm là đi qua cụm, khi đó do các cụm nằm trên đường tròn do đó một đường thẳng đi qua tối đa là hai cụm, mặt khác tổng số cụm là $a+a=2a$ do đó số đường thẳng ít nhất thỏa mãn là $2a/2=a$

Bổ đề 2: Hai điểm cùng màu bất kì trong các điểm đã cho có thể được cùng nằm trong một miền tạo bởi hai đường thẳng sao cho miền chứa hai điểm cùng màu đó ko chứa bất kì điểm khác màu nào còn lại

Chứng minh:
Xét hai điểm $A,B$ cùng màu giả sử là Xanh,  nối $A,B$ xét ở hai nửa mặt phẳng bờ $AB$ có hai điểm $X,Y$ màu đỏ sao cho $X$ gần $AB$ nhất và $Y$ gần $AB$ nhất ($X,Y$ ở hai nửa mặt phẳng khác nhau bờ $AB$) khi đó vẽ đường đường thẳng $//AB$ mà ta gọi là $s$ sao cho khoảng cách từ $s$ đến $AB$ nhỏ hơn khoảng cách từ $X$ đến $AB$ ($X$ không nằm trên $AB$ do ko có ba điểm thẳng hàng, lập tương tự về nửa mp bên kia $s'$ với $Y$

Từ đó $A,B$ nằm trong miền giữa hai đường thẳng $s,s'$ không chứa bất kì điểm đỏ nào khác, đpcm

Quay lại bài toán ở đây là $2013$ đỏ và $2014$ xanh, rõ ràng việc thêm một điểm xanh vào thì bổ đề trên kia vẫn đúng, như vậy số đường ít nhất bằng $\left\lfloor\dfrac{2013+2014}{2}\right\rfloor=2013$
Ta cm $2013$ đường thỏa mãn

Thật vậy xét bao lồi tất cả các điểm trên

TH1: Trong bao lồi (hay gọi tạm là đa giác lồi) tồn tại một đỉnh là đỉnh đỏ gọi là $C$

Khi đó xét hai đỉnh láng giềng với đỉnh đỏ đó (tức hai đỉnh nối với $C$ bằng cạnh của đa giác bao lồi) gọi chúng là $E,F$

Từ $C$ kẻ đường thẳng $l//EF$

Gọi $d_1$ là khoảng cách từ $EF$ đến $l$, $d_2$ là khoảng cách từ điểm xanh gần $C$ nhất

Khi đó vẽ $l'//l$ và $d(l'->l)<min\{d_1,d_2\}$ khi đó dễ thấy $l'$ chứa toàn điểm đỏ và nó chứa ít nhất một đỉnh đỏ

Do đó còn lại $2012$ đỉnh đỏ khác, áp dụng bổ đề 2 cho từng cặp đỉnh ta thu được $\dfrac{2012}{2}.2=2012$ đường

Cộng thêm $l'$ ta có tất cả $2012+1=2013$, chú ý do các đỉnh đỏ được cô lập và các miền chứa điểm đỏ ko chứa bất kì điểm xanh nào nên đó là một cách chia thỏa mãn, đpcm

TH2: Trong bao lồi, không có đỉnh đỏ tức là đỉnh của bao lồi toàn xanh

Khi đó gọi hai đỉnh xanh liên tiếp là $M,N$ và hai đỉnh láng giềng (với $M,N$ tương ứng) $U,V$
Gọi $d_1$ là khoảng cách từ $U$ đến $MN$, $d_2$ là từ $V$ đến $MN$ $d_3$ là khoảng cách của đỉnh đỏ gần nhất đến $MN$
Khi đó vẽ đường thẳng $k//MN$ và có khoảng cách đến $MN$ đủ nhỏ để nhỏ hơn $min\{d_1,d_2,d_3\}$ khi đó ở nửa mp bờ $k$ không chứa $U,V$ thì chứa toàn điểm xanh và chứa ít nhất 2 điểm xanh

Còn $2014-2=2012$ điểm xanh, thực hiện tương tự với bổ đề 2 ta có số đường là $\dfrac{2012}{2}.2+1=2013$ đây là đpcm

Vậy số đường ít nhất là $2013$

 

P/S bài 6 bộ sắp thứ tự $(x,y)$ mình tưởng nó có nghĩa là $(x,y)$ khác $(y,x)$ chứ nhỉ?




#437864 IMO 2013

Gửi bởi nguyenta98 trong 24-07-2013 - 17:26

Bài 2. Trên mặt phẳng cho $2013$ điểm màu đỏ và $2014$ điểm màu xanh, trong đó không có ba điểm nào thẳng hàng. Ta chia mặt phẳng bởi các đường thẳng (không đi qua bất kì điểm nào trong các điểm đã cho) thành các vùng, sao cho không có bất kì vùng nào chứa các điểm có hai màu khác nhau. Hỏi cần ít nhất là bao nhiêu đường thẳng để luôn thực hiện được cách chia đó ?

 

Bài 1 lần này khá đơn giản, ý tưởng dùng quy nạp bổ dọc sau đó bổ ngang, (quy nạp mạnh dạng bảng)

Ý tưởng chung là quy nạp và rất giống như IMO 2012
Các bạn hãy cm hai nhận xét sau đây, khi đó bài toán sẽ dễ dàng được giải
NX1: $(i,k-1)$ đúng thì $(2i,k)$ đúng với $i\geq 1$
NX2: $(i+1,k-1)$ đúng thì $(2i+1,k)$ đúng với $i\geq 1$
Cm hai nhận xét này khá đơn giản, từ đó bài toán được giải xong bằng quy nạp mạnh

 

Bài 2: Dự đoán là $\left\lfloor\dfrac{2013+2014}{2}\right\rfloor$, giờ chỉ cần cm cái này thỏa mãn, chắc là quy nạp, và đã cm được $\left\lfloor\dfrac{2013+2014}{2}\right\rfloor$ là nhỏ nhất, thật vậy xét một cấu hình $2013+2014$ đỉnh như sau: Cho tất cả $2013+2013$ đỉnh đỏ và xanh này lên đường tròn , cách đều nhau, sao cho cứ một điểm xanh lại có một điểm đỏ đối xứng với nó qua tâm, xanh đỏ xen kẽ nhau, như vậy sẽ lấp đủ $2013+2013$ đỉnh, còn thừa một đỉnh xanh ta cho tùy ý vào vị trí giữa hai đỉnh kề nhau bất kì trong cấu hình trên, khi đó dễ thấy không có ba điểm thẳng hàng, số đường ít nhất là $\left\lfloor\dfrac{2013+2014}{2}\right\rfloor$, giờ cm nó là số thỏa mãn là xong